Answer (A) is correct . To determine the profit under the new pricing policy, the sales volume under the old policy must be calculated. $10X – $6X – $12,000 = 0.1($10X) X = 4,000 units The expected volume at the new price of $9 is 5,000 units (4,000 × 125%), and the new profit is $3,000 [(5,000 × $9) – (5,000 × $6) – $12,000].
Answer (B) is incorrect because The sales volume under the old policy is 4,000 units [$10X – $6X – $12,000 – .1($10X)]. The sales will increase 25% when the price is lowered to $9; thus, the new volume is 5,000 units (4,000 × 125%) and the profit would be $3,000. Answer (C) is incorrect because The sales volume under the old policy is 4,000 units [$10X – $6X – $12,000 – .1($10X)]. The sales will increase 25% when the price is lowered to $9; thus, the new volume is 5,000 units (4,000 × 125%) and the profit would be $3,000. Answer (D) is incorrect because The sales volume under the old policy is 4,000 units [$10X – $6X – $12,000 – .1($10X)]. The sales will increase 25% when the price is lowered to $9; thus, the new volume is 5,000 units (4,000 × 125%) and the profit would be $3,000.
|